IMO 1994

------
 
 
Problem A1

Let m and n be positive integers. Let a1, a2, ... , am be distinct elements of {1, 2, ... , n} such that whenever ai + aj ≤ n for some i, j (possibly the same) we have ai + aj = ak for some k. Prove that:

    (a1 + ... + am)/m ≥ (n + 1)/2.

 

Solution

Take a1 < a2 < ... < am. Take k ≤ (m+1)/2. We show that ak + am-k+1 ≥ n + 1. If not, then the k distinct numbers a1 + am-k+1, a2 + am-k+1, ... , ak + am-k+1 are all ≤ n and hence equal to some ai. But they are all greater than am-k+1, so each i satisfies m-k+2 ≤ i ≤ m, which is impossible since there are only k-1 available numbers in the range.

 


 

35th IMO 1994

© John Scholes
jscholes@kalva.demon.co.uk
18 Nov 1998
Last corrected/updated 25 Aug 03